Középiskolai Matematikai és Fizikai Lapok
Informatika rovattal
Kiadja a MATFUND Alapítvány
Már regisztráltál?
Új vendég vagy?

Fórum: "ujjgyakorlatok"

  [1]    [2]    [3]    [4]    [5]    [6]    [7]    [8]    [9]    [10]    [11]    [12]    [13]    [14]    [15]    [16]    [17]    [18]    [19]    [20]    [21]    [22]    [23]    [24]    [25]    [26]    [27]    [28]    [29]    [30]    [31]    [32]    [33]    [34]    [35]    [36]    [37]    [38]    [39]    [40]  

Szeretnél hozzászólni? Jelentkezz be.
[385] Lóczi Lajos2005-10-29 22:49:03

\frac{-11 \pm  3i   {\sqrt{11}}}{2} például megoldás, ahol i a komplex képzetes egység.

Előzmény: [384] lorantfy, 2005-10-29 22:18:31
[384] lorantfy2005-10-29 22:18:31

A racionális számok körében sincs. :-)

Helyettesítéssel kellene visszavezetni másodfokúra!

Előzmény: [383] Lóczi Lajos, 2005-10-29 22:01:15
[383] Lóczi Lajos2005-10-29 22:01:15

Az egész számok körében nincs megoldás :)

Előzmény: [382] lorantfy, 2005-10-29 11:54:41
[382] lorantfy2005-10-29 11:54:41

76. feladat: Oldjátok meg az egyenletet:

x^2+\frac{25x^2}{(x+5)^2}=11

[381] Róbert Gida2005-10-29 02:12:14

Sokkal egyszerübben is meg lehet oldani ezeket, ha lnko(p,q)>1 most itt x59+y59=3, feltehető a szimmetria miatt, hogy x\geq0>y, továbbá x59=3+(-y)59 miatt, hogy x>-y, de akkor x59+y59\geqx59-(x-1)59, de ez x-ben nyilván monoton nő, x=0;1-re nincs megoldás és x\geq2-re x59+y59\geq259-1, így nincs megoldás.

Ez általánosítható arra az esetre is, ha d=lnko(p,q)>1, használjuk, hogy a szomszédos d-edik hatványok különbsége monoton nő,mint az előbb és ha x ismert, akkor az egyenletből abs(y) is megkapható, így az egyenletnek véges sok megoldása lehet legfeljebb.

Az általános esetben feltehető, hogy p vagy q páratlan , különben lnko(p,q)\geq2 teljesül. Legyen például q páratlan, ekkor az egyenlet átírható ( az új y legyen a régi -y ).Az egyenlet

xp-yq=k

, ahol k nem nulla, akkor az egyenletnek minden k-ra véges sok megoldása van, ez S. S. Pillai sejtése, máig megoldatlan.

Előzmény: [373] nadorp, 2005-10-27 09:55:27
[380] Lóczi Lajos2005-10-28 23:19:38

Igen, ezek a számok pár napja jó ismerőseim. :) (Az is érdekes kérdés még, hogy a minimális cáfolómodulushoz tartozó "alkalmas n-ek" halmaza micsoda, hányelemű, stb., de ez nyilván bonyolultabb kérdés lenne.)

Előzmény: [379] Ali, 2005-10-28 14:46:34
[379] Ali2005-10-28 14:46:34

Mégegyszer átolvasva minmod(p,q) definícióját, és "alkalmas n"-hez keresve a minimális modulusokat ez jön ki:

minmod(3,3)=7

minmod(5,5)=11

minmod(7,7)=29

minmod(11,11)=23

minmod(13,13)=53

minmod(17,17)=103

minmod(19,19)=191

minmod(23,23)=47

minmod(29,29)=59

minmod(31,31)=311

minmod(37,37)=149

minmod(41,41)=83

minmod(43,43)=173

minmod(47,47)=283

Ebből valóban adódik a 2/a sejtés.

Előzmény: [378] Ali, 2005-10-28 12:00:54
[378] Ali2005-10-28 12:00:54

Kicsit zavaros ez a minmod(p,q).

Pl.: x5+y5=7 egyenlet megoldhatságát az m=11 modulus cáfolja, míg x5+y5=13 -t m=25, az x5+y5=23 -t pedig m=31.

Akkor most melyik a minmod ?

Előzmény: [377] Lóczi Lajos, 2005-10-27 22:56:47
[377] Lóczi Lajos2005-10-27 22:56:47

Tekintsük tehát az alábbi (*)

xp+yq=n

nemlineáris diofantoszi egyenletet, ahol p>2 és q>2 adott prímszámok, n valamely adott egész szám, és x, y a keresendő ismeretlen egészek. Ennek megoldhatatlanságát szeretnénk kimutatni bizonyos p, q kitevők mellett, alkalmas n jobboldalak esetén.

A (*) egyenlet nyilván megoldhatatlan, ha találunk egy alkalmas m\ge2 egész modulust, hogy (*) megoldhatatlan modulo m.

Gyakorlati szempontból az a fontos, ha minél kisebb ilyen m "cáfoló modulust" sikerül találnunk. Ezért pl. az alábbi számot szeretnénk meghatározni:

minmod(p,q) := min{m\inN| m\ge2 és van olyan n\in{1,2,...,m}, hogy minden x\in{1,2,...,m} és minden y\in{1,2,...,m} esetén (*) NEM teljesül modulo m }.

1. sejtés. a.) Legyenek p\neq 2-nél nagyobb prímek, úgy, hogy 2pq+1 is prím. Ekkor minmod(p,q)=2pq+1.

1. b.) Az összes "cáfoló modulus" minmod(p,q) pozitív egész többszöröse.

2. sejtés. a.) Ha p=q>2 prím, akkor minmod(p,p)=L.p+1, ahol L az a legkisebb pozitív páros szám, amire L.p+1 prím.

2. b.) Az 1. sejtés b.) része a p=q esetben nem igaz.

1. megjegyzés. Bonyolultabbnak tűnik a helyzet, ha a p vagy q számok összetettek is lehetnek.

2. megjegyzés. A 2.a.) sejtés analogonja nem tűnik igaznak a p\neq esetben: p=3 és q=19 esetén pl. minden m\le699 esetén megoldható az eredeti (*) egyenlet, akármilyen egész szám is az n jobboldal.

[376] Lóczi Lajos2005-10-27 21:57:19

De igenis "ujjgyakorlatok", programozási ujjgyakorlatok és "keress szabályszerűségeket a gép outputjában"-ujjgyakorlatok :) -- amíg nincs bizonyítás, addig biztosan.

Előzmény: [375] Csimby, 2005-10-27 20:59:34
[375] Csimby2005-10-27 20:59:34

Talán nyithatnátok egy új topic-ot a témának, mert már nem igazán ujjgyakorlatok :-)

Euler sejtés: Ha s\ge2 és x1,...,xs,z,n pozitív egészek és x1n+x2n+...+xsn=zn, akkor s \ge n.

Ha ez igaz lenne, akkor következne belőle a Fermat-sejtés vagy Wiles-tétel ;-) is, azonban 1966-ban L.J. Lander és T.R. Parkin amerikai matematikusok találtak egy ellenpéldát (n=5,s=4):

a5+b5+c5+d5=1445

Keressük meg a megfelelő a,b,c,d pozitív egészeket. Az n=4,s=3 esetre az előbb említett két úr és J.L. Selfridge már 1967-ben megmutatták, hogy a x14+x24+x34=z4 egyenletnek biztosan nincs olyan pozitív egész megoldása, ahol x1,x2,x3,z mindegyike kisebb, mint 220000. Az első ellenpéldát Noam Elkies adta és megegyezik a 74. feladattal ([341]. hozzászólás). A legkisebb ismert ellenpéldát (n=4,s=3 esetén) R.Frye adta, és a [345]-ös hozzászólásban szerepel.

[374] Lóczi Lajos2005-10-27 20:05:36

Igen nekem is ezek jöttek ki minimális cáfolómodulusnak; esetleg valami sejtés általában? :) Ha lesz időm, beírom az én 2 apró megfigyelésem.

Előzmény: [373] nadorp, 2005-10-27 09:55:27
[373] nadorp2005-10-27 09:55:27

Az első esetben mod(311), a másodikban mod(709) jónak tűnik

Előzmény: [372] Lóczi Lajos, 2005-10-26 23:28:22
[372] Lóczi Lajos2005-10-26 23:28:22

Elkezdett foglalkoztatni, hogy hogyan lehet olyan modulusokat keresni, amelyek azt mutatják, hogy egy

xp+yq=n

alakú diofantoszi egyenletnek nincs megoldása. Mivel egy egysoros programmal sikerült jelentősen automatizálni az egyenletek átvizsgálását, több sejtést is kialakítottam. Ezeket hamarosan beírom ide. Addig is, álljon itt két újabb egyenlet, amelyek bizonyos szempontból a "legrosszabbul" viselkednek (minimális fokszám mellett a legnehezebb cáfolni a megoldásuk létét):

Oldjuk meg az egész számok körében az

x31+y31=31

egyenletet. Ugyanez a kérdés vonatkozik az

x59+y59=3

egyenletre.

[371] Lóczi Lajos2005-10-25 00:21:04

Megtaláltam, amire utaltál:

413=403+173+23=333+323+63,

de a 41 eme tulajdonsága hogyan kapcsolódik (ha egyáltalán) az x4+y5=1728 egyenlethez? Hiszen ezt az egyenletet úgy csináltam, hogy olyan legyen, hogy csak viszonylag "nagy" modulussal lehessen cáfolni, de az 1728 helyett a jobb oldalon nyugodtan lehetne 6 is (mivel ez kongruens 1728-cal modulo 41). Az 1728-nak számomra tehát semmi köze a 41-hez, csak azért választottam, hogy hamis asszociációkat keltsek. A mondandódból viszont az tűnik ki, mintha az 1728-ból jutott volna valahogy eszedbe a 41, mint cáfoló modulus. Ez vajon véletlen, vagy van valami a háttérben?

Előzmény: [368] Káli gúla, 2005-10-24 23:03:26
[370] Lóczi Lajos2005-10-24 23:11:17

De azt nem látom, hogy mi a kapcsolat a két egyenlet (azaz a 4.-5. hatványos és a köbösszeges) között? Mire lehet következtetni a másikra nézve, ha tudjuk, hogy az egyiknek mi a megoldása/nincs megoldása?

Előzmény: [368] Káli gúla, 2005-10-24 23:03:26
[369] Káli gúla2005-10-24 23:06:03

A "123-höz hasonlóan" szövegrész törlendő. Elnézést.

[368] Káli gúla2005-10-24 23:03:26

Egyrészt az x4+y5=1728 egyenletnek nincs megoldása mod 41, másrészt, a 123-höz hasonlóan a 413 is többféleképpen írható fel, de nem két, hanem három köbszám összegeként.

Előzmény: [366] Lóczi Lajos, 2005-10-24 22:50:25
[367] jonas2005-10-24 22:59:46

Valóban, fel is van sorolva a listában (ami nem rendezett).

Talán kiszámolom, ha kevésbé vagyok fáradt.

Előzmény: [363] Lóczi Lajos, 2005-10-24 22:20:20
[366] Lóczi Lajos2005-10-24 22:50:25

Miért, az x3+y3+z3=413 egyenletről mit lehet tudni? (És hogyan kapcsolódik az x4+y5=1728-hoz? Kíváncsi lettem.)

Előzmény: [364] Káli gúla, 2005-10-24 22:35:27
[365] nadorp2005-10-24 22:36:24

Ez poén, beismerem ezzel a konstrukcióval még nem találkoztam.

Előzmény: [360] jonas, 2005-10-24 21:20:47
[364] Káli gúla2005-10-24 22:35:27

De épp a 40-nél kár lenne abbahagyni a keresést. Erre próbáltam tegnap is célozgatni, a 413=x3+y3+z3 egyenlet megoldásszámával.

Előzmény: [363] Lóczi Lajos, 2005-10-24 22:20:20
[363] Lóczi Lajos2005-10-24 22:20:20

Sajnos a megoldásod hibás: az első számkupacban ott az 1-es, míg a másodikban a 7-es, és 1+7=8.

Előzmény: [362] jonas, 2005-10-24 21:49:44
[362] jonas2005-10-24 21:49:44

Jé, tényleg elrontottam valamit.

A 40 modulus jó: x4mod 40 lehet 0, 1, 16, 25

y5mod 40 lehet 0, 1, 32, 3, 24, 5, 16, 7, 8, 9, 11, 13, 15, 17, 19, 21, 23, 25, 27, 29, 31, 33, 35, 37, 39

Így aztán x4+y5mod 40 lehetséges értékei 0, 1, 32, 3, 24, 5, 16, 7, 8, 9, 11, 13, 15, 17, 19, 21, 23, 25, 27, 29, 31, 33, 35, 37, 39, 2, 4, 6, 10, 12, 14, 18, 20, 22, 26, 28, 30, 34, 36, 38.

1728mod 40=8 nincs ezek között.

Előzmény: [361] Lóczi Lajos, 2005-10-24 21:21:54
[361] Lóczi Lajos2005-10-24 21:21:54

Pedig lehet. Segítség: cáfoljuk meg a megoldás létezését, hogy az egyenletet egy alkalmas maradékosztály felett nézzük.

Előzmény: [359] jonas, 2005-10-24 21:12:05

  [1]    [2]    [3]    [4]    [5]    [6]    [7]    [8]    [9]    [10]    [11]    [12]    [13]    [14]    [15]    [16]    [17]    [18]    [19]    [20]    [21]    [22]    [23]    [24]    [25]    [26]    [27]    [28]    [29]    [30]    [31]    [32]    [33]    [34]    [35]    [36]    [37]    [38]    [39]    [40]